Midterm Cherry Questions

Ace your homework & exams now with Quizwiz!

*Chapter 19 55. Nursing's primary concern related to the appropriate use of social media is to preserve: A. confidentiality. B. professionalism. C. cultural sensitivity. D. effective communication.

A. Confidentiality Reason: The nurse must be acutely aware of the need to preserve the client's privacy and the confidentiality regarding client information. The nurse must also be aware of the consequences of failing to do so effectively. REF: pp. 344

*Chapter 17 5. Which of the following activities engaged in by a nurse manager would be expected to have the greatest impact on patient care? a. Role modeling effective communication between all levels of staff b. Conducting a survey to identify factors affecting staff job satisfaction c. Providing all nursing staff with opportunities to influence unit policies d. Providing all staff members with written copies of organizational policies

A. Role modeling effective communication between all levels of staff Reason: Positive role modeling is an effective tool the nurse can use to create a positive team spirit and promote high-quality patient care. Positive role modeling simply means that the nurse performs the job in such a way that he or she demonstrates ideal performance as a professional nurse; others hopefully will follow the example. REF: pp. 306-307

Chapter 17 3. Which skill is most important for a nurse to possess when attempting to exercise or earn informal power within an organization?

Ability to persuade others Reason: Informal leadership is exercised by the person who has no official or appointed authority to act, but can persuade and influence others. The informal leader may have considerable power in the work group and can influence the group's attitude and significantly affect the efficiency and effectiveness of workflow, goal setting, and problem solving. REF: p. 297-298

*Chapter 17 1. According to the definition, which word best describes leadership? A. Allocate B. Influence C. Evaluate D. Control

B. Influence Reason: Leadership is the ability to guide or influence others. REF: p. 296-297

*Chapter 19 52. The most important concept to remember when using both verbal and nonverbal communication is that: A. people are more likely to accept verbal messages than nonverbal ones. B. nonverbal messages are accepted as true more often than verbal ones. C. touch as a nonverbal form of communication should be avoided. D. avoiding eye contact is viewed as being untruthful by all cultures.

B. nonverbal messages are accepted as true more often than verbal ones. Reason: An important concept to remember is that when the verbal message and the nonverbal message do not agree, the receiver is more likely to believe the nonverbal message since body language is often the most trusted indicator for conveying feelings, attitudes, and emotions. REF: p. 333

*Chapter 6 92. The primary reason nurses engage in research is to: A. achieve equal status with other health care sciences. B. provide the most appropriate evidence-based client care. C. advance the profession of nursing in the eyes of the public. D. collaborate with other members of the science community.

B. provide the most appropriate evidence-based client care . Reason: The main outcome of a research activity for a nurse is to put the knowledge gained to work in health care delivery. Although engaging in research may achieve the other outcomes, they are not the primary reason behind nursing research. REF: p. 90-91

*Chapter 16 202. In the case of a terrorist attack that employs an explosive agent, medical personnel including first responders must be prepared to manage numerous victims with: A acute poisoning. B thermal burns. C shrapnel injuries. D contamination risks.

B. thermal burns Reason: Explosive agents are capable of producing high degrees of temperature resulting in damaging thermal burns. Chemical and biologic agents often require decontamination because of the risk of contamination of others. Radioactive agents can result in acute poisoning while nuclear explosions have a high risk of shrapnel injuries. REF: p. 284, Table 16-1

*Chapter 8 33. Which of the following situations fails to meet the criteria for establishing nursing negligence or malpractice? a. A nurse comes to work under the influence of alcohol. b. The nurse leaves a client's bed in the raised position, resulting in a fall. c. The nurse fails repeatedly to document a client's response to pain medication. d. A nurse assigns first-time ambulation of a post-op client to an aide and the client falls.

C. The nurse fails repeatedly to document a client's response to pain medication. Reason: The criteria require that the nurse-client contract be broken such as the alcohol scenario or preventable injury to the client as a result of the failure to follow good nursing practice. REF: pp. 133

Chapter 17 6. When considering the philosophy and values of a health care facility, _____________________ is the highest priority.

Compassion Reason: Compassion is the highest priority when providing care and comfort to people in need with respectful and dignified treatment at all times. REF: p. 303, Box 17-4

*Chapter 4 164. Where can a registered nurse (RN) find the most reliable guidelines regarding the appropriate delegation of activities or tasks to unlicensed nursing personnel? A. The National Council of State Boards of Nursing (NCSBN) B. The American Nurses Credentialing Center (ANCC) C. The American Nurses Associate (ANA) D. That state's nurse practice act

D. That state's nurse practice act Reason: Guidelines for delegation have been developed by many nursing organizations, including the ANA and NCSBN. Although the professional organizations' guidelines are helpful, it is the nurse practice acts of individual states that establish the legal definitions of appropriate delegation practices. Because regulations differ among states, each nurse must identify and understand the regulations for the state in which he or she practices. The ANCC is an organization that focuses on the certifications of nurses in various areas of nursing practice. REF: p. 67

Chapter 17 7. The __________________ management style encourages staff involvement in the decision-making process.

Democratic / Participative Reason: The Democratic/Participative manager encourages staff participation in decision making. Involves staff in planning and developing new ideas and programs while believing in the best in people. This manager communicates effectively, and provides regular feedback and so builds responsibility in people. Works well with competent, highly motivated people. REF: p. 300, Box 17-3

Chapter 17 2. A nurse manager will likely have which of the following types of power?

Legitimate Reason: Legitimate power is based on an official position in the organization. Through legitimate power, the manager has the right to influence staff members, and staff members have an obligation to accept that influence. REF: p. 297

Chapter 17 4. Which activity demonstrates the organizational Chaos Theory?

Planning for unexpected staffing shortages Reason: The Chaos Theory suggests that a degree of order can be attained by viewing complicated behaviors and unpredictable situations as predictable by planning for them. REF: pp. 301

Chapter 26 174. When discussing hospital-based long-term job satisfaction with a group of senior-level nursing students, a nursing administrator shares that:

a. "I'd suggest you get a variety of nursing experiences before deciding on your dream job." Reason: If the choice of the clinical setting has been based on experiences as a student, the new graduate needs to be prepared to have different perceptions in a new role. At a minimum, experiences that are highly enjoyable on the limited-time basis of a student schedule may feel different when the new graduate functions in that role full time. It also is good to have a mix of experiences and learning opportunities before making a definitive decision. REF: p. 455

Chapter 28 192. Which statement made by a novice nurse demonstrates an understanding of the computer-adaptive testing (CAT) methods utilized when taking the NCLEX-RN examination?

a. "I'm successful when I've earned a 95% chance of reaching the passing standard." Reason: It is important for students to understand that the goal of CAT is to determine competency based on the difficulty of questions answered correctly rather than the number of questions answered correctly. The CAT is individualized, and two people taking the examination at the same time will not have the same questions. The other options are not correct. REF: p. 488

Chapter 20 79. Which statement made by the RN demonstrates a lack of effective delegation to a LPN? (Select all that apply.)

a. "If you begin that complex dressing change while I'm off the floor, get someone to help you." b. "Remember the patient is depressed so don't say anything to make her more sad." Reason: Not being available to provide effective supervision and addressing the issue in such a general manner is not a demonstration of effective delegation. Assigning a patient whose emotional state is unstable is not reflective of effective delegation. Evaluating the staff member's competency to perform a task or care for a patient with a particular set of needs and the requirement of mandatory competency testing is a factor in effective delegation. It is necessary to assure that the staff member has the resources including supplies to accomplish the assigned task. REF: p. 360, Box 20-4

Chapter 27 188. What topic is appropriate for the nursing position interviewee to address during an interview with a recruiter? (Select all that apply.)

a. "Is there a pay differential for shift work?" b. "What is the process for earning a pay increase?" c. "How will my job performance be evaluated?" d. What will my salary be during the orientation period?" Reason: The recruiter will be prepared and qualified to provide information regarding pay differentials and salaries during the orientation period, as well as the evaluation processes for pay raises and position advancements. Exactly when pay raises will occur is dependent on specific institution policies and the results of the evaluation process and so is a question the recruiter is not prepared to answer. REF: p. 479, Boxes 27-7 and 27-8

Chapter 20 72. Which statement made by an unlicensed assistive personnel (UAP) would cause the registered nurse team leader the most concern?

a. "It's good to know that the nurse is really the one responsible for the client's care." Reason: The UAP is responsible for his or her own actions. A belief that the nurse is fully responsible is a concern and needs immediate attention by the nurse. It is true that the nurse will follow up and evaluate the client's care. The statements concerning being new and asking for help are appropriate. REF: p. 354

Chapter 19 54. When a client expresses doubt whether a complicated procedure will actually help his condition, the nurse best avoids blocking the communication by responding:

a. "What makes you doubt that your condition will benefit from having the procedure done?" Reason: By opening the communication up to further discuss the client's concerns, the nurse has validated the client's feelings, acknowledged his concerns, and provided the opportunity for further communication. REF: p. 334

Chapter 8 40. Nonpayment hospital acquired conditions include: (Select all that apply.)

a. A Stage III pressure ulcer b. Hypoglycemic coma c. Catheter associated urinary tract infection (UTI) d. Vascular catheter associated infection Reason: Conditions that develop as a result of a hospitalization and but reimbursed for include State III pressure ulcers, critical conditions resulting from poor glycemic control, and infections resulting from the mismanagement of invasive interventions (urinary and vascular catheters). Stress-related hypertension is not a condition that results from mismanagement. REF: p. 123, Box 8-1

Chapter 9 41. Which of the following scenarios demonstrates a nurse engaging in value clarification?

a. A nurse reflects on the question, "What would be important to me if I was told my cancer was terminal?" Reason: The process of values clarification attempts to examine the values one holds and how each of those values functions as part of a whole. Nurses must acknowledge their own values by considering how they would act in a particular situation. REF: pp. 167

Chapter 28 196. What actions are taken by the state board of nursing to best achieve the goals of the NCLEX-RN examination? (Select all that apply.)

a. A practice analysis is conducted every 3 years b. The NCLEX focuses on the activities novice nurses regularly perform c. Novice RNs are on the NCLEX review panel d. All those taking the examination must be graduates of approved schools of nursing Reason: The goals of the NCLEX-RN examination are to safeguard the public from unsafe practitioners and to assist state board of nursing in determining candidates' capabilities for performing entry-level RN positions. This is accomplished by updating the NCLEX test blueprint every 3 years with the input of novice nurses in order to reflect the skills novice nurses are expected to possess in order to practice safely. Only those who are graduates of an approved nursing program are eligible to take the test in order to assure access to the necessary nursing knowledge. The test has been computerized so that current measurement theories and technologies can be used to evaluate the results of the testing process. REF: pp. 487-488

Chapter 28 200. NCLEX-RN adheres to an organizational framework that focuses on client needs in areas that include: (Select all that apply.)

a. A safe and effective environment b. The promotion and maintenance of health c. Psychosocial integrity d. Physiologic integrity Reason: The care areas included are: a safe and effective environment; the promotion and maintenance of health; psychosocial integrity; and physiologic integrity. Financial concerns are not a care area. REF: p. 491-492

Chapter 4 168. The following nursing associated initials with their appropriate description.

a. APN i. 3- Legal title for nurses prepared by education and competence to perform independent practice b. CCNE i. 2- Responsible for accrediting baccalaureate and higher-degree nursing programs c. ANA i. 4- Professional organization that represents all registered nurses d. NCSBN i. 1- Organization whose membership consists of the board of nursing of each state or territory

Chapter 6 96. A novice nurse reading a research article should first concentrate on which sections of the article?

a. Abstract and discussion Reason: The abstract section of the article gives an overview of the study, and the discussion section offers suggestions for nursing practice based on the findings of the research study. These two sections often are the easiest for the novice to interpret. REF: p. 98-99

Chapter 25 70. What are the characteristics of SMART goals (Select all that apply.

a. Achievable b. Manageable c. Reasonable Reason: SMART goals are specific, measurable, reasonable, and time based. REF: p. 440-441

Chapter 4 169. Which of the following is an area of clinical specialist nursing for which the American Nurses Credentialing Center (ANCC) offers a certifying examination? (Select all that apply.)

a. Adult Health b. Pediatrics c. Adult Psychiatric and Mental Health Reason: ANCC offers Clinical Nurse Specialist certifying examinations for Adult Health CNS, Diabetes Management—Advanced, Adult Psychiatric and Mental Health CNS, Child Adolescent Psychiatric and Mental Health CNS, Gerontologic CNS, Pediatric CNS, and Public Community Health CNS. Home Health and Informatics Nursing are specialties that the ANCC offers certifying examinations for. REF: p. 72, Box 4-1

Chapter 9 48. Match the following values with their appropriate professional behavior.

a. Altruism i. 2- Expresses concern about issues that have implications for health care b. Human Dignity i. 1- Shows value and respect all patients and colleagues c. Social Justice i. 3- Acts as a health care advocate d. Integrity i. 4- Provides care based on an ethical framework

Chapter 23 14. Which organization represents nurses and addresses health care issues on a national level?

a. American Nurses Association (ANA) Reason: As a special interest group representing the interests of 3.1 million professional nurses throughout the United States, the American Nurses Association (ANA) carries a strong voice and high visibility in influencing health policy and nursing practice. REF: pp. 402 and 406

Chapter 22 82. Which nursing activity best demonstrates the quality management principle of "All One Team?"

a. Asking a wound care team nurse how to best educate a client on wound prevention Reason: Demonstrating faith in the people who comprise the health care team and recognizing the expertise of particular team members reflects the "all one team" principle. Quality is demonstrated by considering clients as those who define whether their care is appropriate and of high value. Knowledge, especially when evidence based, demonstrates the scientific approach to client care. REF: pp. 383-384

Chapter 17 8. What interventions will a nurse manager implement when coaching a newly licensed nurse regarding the way a patient's complaint was handled?

a. Asks, "what will you do differently the next time this situation occurs?" b. Begin the discussion with a personal interpretation of the situation c. Approach the conversation in a nonjudgmental fashion Reason: Effective coaching includes discussing situations in a neutral way—avoid judgmental language that will put the other person on the defensive, encouraging the other person to provide his or her perspective about the situation and to reflect on his or her performance through open-ended questions. Being realistic by focusing on what is actually achievable. REF: p. 307, Box 17-6

Chapter 26 180. The roles of a certified registered nurse anesthetist (CRNA) include: (Select all that apply.)

a. Assisting in the management of pain associated with childbirth b. Managing chronic and acute ventilatory problems c. Providing therapeutic pain blocks d. Providing anesthesia based care Reason: CRNAs provide anesthesia and anesthesia-related care on request, assignment, or referral by a client's physician, most often to facilitate diagnostic, therapeutic, or surgical procedures as well as performing consultation or assistance for management of pain associated with obstetric labor and delivery, management of acute or chronic ventilatory problems, or management of acute or chronic pain through the performance of selected diagnostic or therapeutic blocks. Direct supervision by an anesthesiologist is not required. REF: p. 463

Chapter 8 32. Which nursing action best complies with the expectations for nursing care defined by the "never events" identified by the 2012 Rules of Participation for Hospitals?

a. Attending an in-service on evidence-based practice on urinary catheter care Reason: Nurses are required to develop greater expertise in the provision of evidence-based patient care, case management, and discharge planning in order to avoid "never events." REF: pp. 122

Chapter 19 58. Match the following conflict manage styles with their definition.

a. Avoidance i. 3- One person uses passive behaviors and withdraws from the conflict; neither person is able to pursue goals. b. Accommodation i. 1- One person puts aside his or her goals to satisfy the other person's desires. c. Force i. 2- One person achieves his or her own goals at the expense of the other person. d. Compromise i. 5- Both people actively try to find solutions that will satisfy them both. e. Collaboration i. 4- Both people give up something to get partial goal attainment.

Chapter 23 18. Guidelines supported by the American Nursing Association (ANA) for communicating with government policy members include: (Select all that apply.)

a. Be brief and concise about what you want to communicate b. Be specific about the issue you are communication about c. Be sure to identify yourself by name and address d. Be persistent by using various accepted forms of communication Reason: Effective communication with law makers includes brief, concise, and specific communication about the topic of interest. Being persistent is appropriate but being aggressive is not. Identifying yourself is the initial step in communication. REF: pp. 407-408

Chapter: 24 156. Being able to introduce ideas or values brought from nursing school and integrate them into the work environment is referred to as __________________________.

a. Biculturalism Reason: Biculturalism is the joining of two contradictory value systems, in this context, those of school values with those of the workplace. Biculturalism is designed to enhance a positive self-image and help novice nurses set realistic goals for practice. REF: p. 425

Chapter 10 110. Match the following groups with the appropriate cultural healer.

a. Blacks i. 1- Root doctor b. Asians i. 4- Herbalist c. Hispanics i. 2- Espiritualista d. American Indians i. 3- Shaman

Chapter 10 111. Match the following groups with the behavior that is considered inappropriate.

a. Blacks i. 2- Touching another's hair b. Asians i. 1- Shaking hands with persons of the opposite sex c. Hispanics i. 4- Expression of negative feelings d. American Indians i. 3- Establishing direct eye contact

Chapter 8 36. ___________ occurs when a standard of care is not met by the responsible caregiver.

a. Breach of duty Reason: Breach of duty occurs when one person or company has a duty of care toward another person or company, but fails to live up to that standard. REF: p. 119

Chapter 26 171. Which nursing role is basic to the nursing profession?

a. Care provider Reason: The role of care provider is basic to the nursing profession. Caring is always central to nursing interventions and is an essential attribute of the expert nurse. The other options, while certainly nursing roles, are not the basic function of nursing: caring. REF: p. 451

Chapter 9 46. Fidelity is based on the virtue of _________________.

a. Caring Reason: Fidelity is the agreement to keep promises and commitments, based on the virtue of caring. REF: p. 162

Chapter 1 124. In the Middle Ages (467 BCE to AD 1450), women delivered food, medicine, and care to the community's ill based on the:

a. Christian concepts of charity and the sanctity of human life. Reason: The Middle Ages saw the Christian church as the central figure in the organization and management of health care, and its concepts of charity and sanctity of human life were influential in the practice of caring for the sick. Beginning in prehistoric times, there was a belief that illness was a direct outcome of displeasing the gods, which gave rise to the practice of making offerings to the gods. In many cultures, it is still believed that illness is related to punishment from the god(s). The concept of patient-centered care and use of the scientific method of problem solving originated with the Greeks (1500-100 BCE). The Romans (27 BCE to AD 476) are credited with the concepts of good hygiene and sanitation being vital to good health. REF: p. 6-7

Chapter 1 127. The American Red Cross is established by _________.

a. Clara Barton Reason: The American Red Cross is established by Clara Barton in 1881. REF: p. 46, Box 1-1

Chapter 2 133. The play Miss Evers' Boys was one of the first literary presentations that depicted nurses as:

a. Client Advocates Reason: The play and eventual television movie Miss Evers' Boys tells the story of the now-infamous Tuskegee experiment that focused on the treatment of syphilis among black men between1932 and 1972. Miss Evers was the nurse in charge of recruiting subjects. The play describes how her request for the inclusion of penicillin into the treatment plan was denied, even though penicillin was the treatment of choice for the disease after 1947. REF: p. 26

Chapter 3 147. A nurse can find guidance in managing ethical nursing issues in the ANA's _________________.

a. Code of Ethics for Nurses with Interpretive Statements Reason: The American Nurses Association's Code of Ethics for Nurses with Interpretive Statements (2001) provides a succinct statement of the ethical obligations and duties of every individual who enters the nursing profession. REF: p. 49

Chapter 17 10. Which characteristics are expected to be represented in a health care facility's mission statement? (Select all that apply.)

a. Commitment to professional excellence b. Ethical treatment of all patients and staff c. Teamwork to deliver quality care d. Utilization of innovated service delivery methods Reason: Appropriate philosophies and values include commitment to professional and individual excellence, with support for personal and professional growth, ethical and fair treatment for all through a commitment to relationships based on fairness and trust with our patients and our employees. Teamwork is consistently demonstrated as we work together to provide ever-improving quality care. Compassion is our highest priority; we will always provide care and comfort to people in need with respectful and dignified treatment at all times. Innovation in service delivery is accomplished by investing in the development of new and better ways to deliver health care. REF: p. 303, Box 17-4

Chapter 19 56. Relaying information using words, letters, symbols, or body language is referred to as _________________.

a. Communication Reason: Communication is a process of relaying information between or among people by the use of words, letters, symbols, or body language. REF: p. 329

Chapter 8 38. Which situation frequently results in a nursing negligence claim? (Select all that apply.)

a. Communication b. Reporting c. Monitoring d. Safety Reason: The most frequent allegations of nursing negligence includes failures related to communication, reporting and monitoring patients and patient conditions, and addressing patient safety. The facilitating of patient health (cure) is not a frequent area of negligence. REF: p. 141, Box 8-4

Chapter 7 29. What component is included in the Hospital Consumer Assessment of Health care Providers and Systems survey? (Select all that apply.)

a. Communication with nurses b. Responsiveness of hospital staff c. Pain management d. Cleanliness and quietness of hospital environment Reason: The components of the HCAHPS survey includes question assessing communication with nurses, responsiveness of hospital staff, pain management and the cleanliness and quietness of hospital environment. It does not address the cost of hospital services. REF: p. 114, Box 7-1

Chapter 22 90. Which infections are currently considered a Never Event by Medicare and Medicaid? (Select all that apply.)

a. Coronary artery bypass graft site b. Abdominal incision site associated with bariatric surgery c. Catheter-associated urinary tract infection d. Site of implanted cardiac electronic device (CIED) Reason: Never Events currently include infections associated with surgical site following coronary artery bypass graft; certain orthopedic procedures; bariatric surgery for obesity; cardiac implantable electronic device (CIED). Postcataract surgery infection is not included. REF: p. 392; Box 22-3

Chapter 25 69. Place the following premeeting tasks in the proper order (1 is the first step and 6 is the final step)

a. Create a list of the meeting's topics b. Determine and announce the date, time, and location of meeting c. Determine the meeting's purpose d. Create the meeting's agenda e. Determine who needs to attend the meeting f. Send out a reminder of the meeting Reason: Premeeting tasks are accomplished in this order; determine the purpose of the meeting; create a list of the meeting's topics; determine who needs to attend the meeting; determine and announce the date, time, and location of the meeting; create the meeting's agenda and finally send out a reminder of the meeting. REF: p. 447; Box 25-2

Chapter 22 83. How can a nurse manager best foster high-quality and safe nursing care among the nursing staff?

a. Create a unit culture where asking questions about health care interventions is encouraged Reason: Health care organizations focusing on quality and safety encourage inquiry, making it okay to ask questions and providing resources to access information needed through various means, including informatics. Although praise and incentives are appropriate, they are not effective if the unit culture is not accepting. Placing emphasis on such interventions alone will not be successful if not supported by nursing managers and leaders. REF: p. 396

Chapter 22 87. A ______ is an individual or group who seek a product or service and whose needs and expectations determine the quality of the service.

a. Customer Reason: A customer is an individual or group who relies on an organization to provide a product or service to meet some need or expectation. It is these customer needs and expectations that determine quality.

Chapter 3 146. A nursing class is divided into four teams. The teams are asked to make recommendations for resolving concerns that have a negative impact on the learning environment. Which of the following interventions should be implemented?

a. Defining behaviors that will not be tolerated in the classroom since they do not demonstrate respect or civility. Reason: The decline of respectful, civil interpersonal behavior is a social problem that impacts even the learning environment. The initial defining of unacceptable behaviors is most effective in setting expectations of classroom etiquette. While the other options may represent reasonable requests, they are not as directed toward the primary issue of effective learning environments. REF: p. 47

Chapter 9 49. Match the following ethical theories and principles with their appropriate description.

a. Deontology i. 1-One is bound by moral rules b. Utilitarianism i. 3-The good of the many outweighs the good of the few c. Autonomy i. 4-Personal freedom has priority d. Beneficence i. 5-Stresses compassionate care e. Nonmaleficence i. 2- Duty to avoid doing harm

Chapter 19 59. Place the following activities in the proper sequence in order to best achieve the development of a professional communication style.

a. Developing an effective personal conflict management style b. Avoiding the use of negative communication techniques like cognitive distortions c. Recognize one's personal use of negative communication techniques like blocking d. Adopting the use of "I messages," conveying empathy, and open communication techniques e. Understanding the influence gender, cultural, and age has on the communication process Reason: The first step toward developing a professional communication style is understanding the many complex and varied factors that influence the communication process, such as gender, cultural, generational, and interdisciplinary differences, each of which present many challenges for the nurse who must strive to understand and to be understood. The second step is adopting positive communication techniques, which include developing trust, using "I messages," establishing eye contact, keeping promises, feeling empathy, using an open communication style, clarifying information, and being aware of body language. The third step is for each nurse to reflect on his or her use of negative communication techniques, such as blocking, false assurances, conflicting messages, logical fallacies, and cognitive distortions that may interfere with effective relationships with patients and coworkers. Recognizing—then avoiding—the use of these negative communication techniques is essential if the nurse is to move toward a more professional communication style. The next step in developing a professional communication style is learning to address and resolve conflict in a positive way. REF: pp. 349

Chapter 25 66. Ensuring that the right amount of work matches the right task is a part of ____________________.

a. Energy management Reason: Energy management involves ensuring that the right amount of effort matches the right task to optimize an outcome while gauging the amount of personal energy expended or taxed to achieve the desired result. REF: p. 435-436

Chapter 2 134. A new nurse manager is concerned about the workforce diversity in the unit. A staff survey is created to assess the opinion of those who work on the unit about their perceptions. Which of the following questions should be added to the survey?

a. Do you have the perception that nursing is a female profession? Reason: Nursing continues to be viewed as a female profession although the number of males entering the profession is steadily increasing. The public views nursing as an interesting and meaningful profession that provides good job security but does not provide job independence. The perception of job independence affects both male and females equally. REF: pp. 29-30

Chapter: 24 158. Which self-assessment questions would be helpful when evaluating one's personal sense of self-esteem? (Select all that apply.)

a. Do your good points seem ordinary and your failings all important? b. Do you struggle with feelings of inferiority? Reason: The nurse with high self-esteem will possess a positive attitude; have pride in his or her accomplishments; have a balanced view of personal strengths and weaknesses; are not overly critical of oneself. Feeling that one has earned the rewards of life is an indicator of high self-esteem. REF: p. 428, Box 24-6

Chapter 26 176. The title of_____________ is used to identify an expert in advanced nursing practice at the doctorate level.

a. Doctor of nursing practice Reason: The Doctor of nursing practice (DNP) is an expert in advanced nursing practice who has an earned clinically focused doctorate degree in nursing. REF: p. 450

Chapter 2 140. Which statement accurately describes elements concerning the play Miss Ever's Boys? (Select all that apply.)

a. Dramatizes an ethical nursing dilemma b. Focuses on the infamous Tuskegee experiment c. Involved the administration of penicillin as a treatment modality d. Treatment of syphilis Reason: The play tells the true story of Nurse Rivers, who was hired to recruit and retain young black men into the infamous Tuskegee experiment designed to describe the long-term effects of untreated syphilis. Although the study began in 1932, penicillin became the treatment of choice for syphilis in 1947 in the civilian population. When subjects asked Nurse Evers to obtain the new treatment of penicillin for them and she sought to do so, the physician investigators required her to discourage them from treatment. The play introduces non-nurses to the dilemmas of nursing practice during that era and the consequences of misplaced faith and trust in other health care disciplines. REF: p. 26

Chapter 9 43. Which scenario demonstrates the preconventional level of moral development?

a. Driving within the speed limit in order to avoid getting a speeding ticket Reason: Moral decisions made at this level (preconventional) are simply a response to some threat of punishment. Acting out of respect or a desire to please demonstrates the conventional level, where behavior is less self-centered. REF: pp. 168

Chapter 16 206. In 2014 the epidemic, __________ threatened multiple countries in West Africa

a. Ebola Reason: A more recent world threat is the 2014 Ebola epidemic, affecting multiple countries in West Africa. REF: p. 291

Chapter 26 178. Which characteristics are associated with a certified nurse leader (CNL)? (Select all that apply.)

a. Educated as a nursing generalist b. Coordinate and implement client care c. Evaluate and implement evidence-based practice Reasons: The CNL is a generalist who coordinates and implements evidence-based nursing practice. The clinical nurse specialist (CNS) designs and evaluates patient-specific based programs and is tasked with generating new evidence upon which nursing interventions are based. REF: p. 462-463

Chapter 26 179. What are the traditional duties and responsibilities of the professional nurse regardless of practice setting? (Select all that apply.)

a. Educator b. Advocate c. Counselor d. Care coordinator Reason: Traditional duties and responsibilities of the professional nurse, regardless of practice area or setting, such as the roles of care provider, educator, counselor, client advocate, change agent, leader and manager, researcher, and coordinator of the interprofessional health care team. Informatics is a nursing specialty. REF: p. 450

Chapter 20 77. Match the following actions with the appropriate responsibility regarding the delegation of nursing interventions.

a. Effective communication i. 1- UAP's statement, "I'll notify you of Mr. Smith's BP as soon as I take it so you can decide whether he gets his medication." b. Appropriate supervision i. 3- RN initially observes a newly hired LPN is changing a simple dressing c. Evaluation and reassessment i. 2- RN discusses the way the LPN handled a patient complaint

Chapter 16 210. Which service personnel are traditionally considered first responders when a community disaster occurs? (Select all that apply.)

a. Emergency Medical Technicians b. Law enforcement officers c. Firefighters Reason: Traditionally, first responders to emergencies have been the law enforcement, firefighters, and emergency medical technicians who respond with ambulances. Physicians and nurses are generally involved when individuals make their way to hospital emergency departments. REF: p. 283

Chapter 1 126. Which behavior provides evidence addressing the ability of nursing as a profession to provide high-quality, effective care as patient-centered needs intensify?

a. Enrolling in a Masters of Nursing Education graduate program Reason: The need for nurses to provide the care for the influx of clients projected in the future is best addressed initially through the increase in nurses with graduate degrees. This allows for the education and training of future nurses who are introduced to nursing through effective recruiting targeted at various populations. Health screening is directed toward the early detection of health care needs. Political activism is directed toward health care policy change and enactment on national and even local government levels. REF: p. 18-19

Chapter 4 170. What characteristic applies to process of licensure? (Select all that apply.)

a. Establishes minimal levels of practice b. Is granted by governmental legislation c. Administered by state associated boards Reason: Licensure establishes minimal levels of practice, whereas certification recognizes excellence in practice and is granted and governed by legislation and administered through the state board of nursing. Certification recognizes excellence in practice and is awarded by nongovernmental agencies. REF: p. 71

Chapter 10 107. The belief that one's own culture is superior to all other cultures is called _________________.

a. Ethnocentrism Reason: Ethnocentrism is the belief that one's own ethnic group, culture, or nation is best. REF: p. 180

Chapter 16 208. Match the following steps of response to an active shooter with an example of an acceptable action.

a. Evacuate (RUN) i. 2- Do not attempt to move the wounded b. Hide Out (HIDE) i. 3- Remain quiet c. Take action against the active shooter (FIGHT) i. 1- Commit to your planned actions

Chapter 6 98. Currently, __________ is the major driving force in the delivery of health care.

a. Evidence-based practice (EBP) Reason: In this second decade of the twenty-first century, evidence-based practice (EBP) has become a major driving force in the disciplinary life of clinicians, students, educators, administrators, and policymakers. REF: p. 95-96

*Chapter 8 39. Which incidences are the basis for disciplinary action by state boards of nursing? (Select all that apply.) A. Falsification of nursing records B. Medicare fraud C. Misappropriation of a patient's personal belongings D. Practicing without a valid license E. History of employment absenteeism

a. Falsification of nursing records b. Medicare fraud c. Misappropriation of a patient's personal belongings d. Practicing without a valid license Reason: The state boards of nursing will act upon reports of the falsification of records, stealing from patients, engaging in Medicare fraud, and practicing without a valid license. A history of absenteeism is an employer issue. REF: p. 128, Box 8-2

Chapter 25 67. Match the term with the appropriate description.

a. Goal i. 2- An attainable act in a specific period of time b. Objective i. 3- A measurable and identifiable act that implements a desired action. c. Milieu i. 1- The social or physical setting of an event

Chapter: 24 159. What guidelines should a novice nurse consider when selecting a mentor? (Select all that apply.)

a. Has experience in your nursing specialty b. Someone you can easily establish regular contact with c. Possesses characteristics you view yourself being weak in Reason: A mentor who lives nearby or works in the same facility will make interaction easy to arrange. Having experience in the field of nursing you share will help with clinical situations. By having the strengths you lack, the mentor can certainly be a role model. Similar age and educational background are not necessarily advantages nor is having known the person for a length of time. REF: p. 426, Box 24-5

Chapter 23 16. A course of action undertaken by governments or health care organizations to achieve a particular health outcome is referred to as a _________________.

a. Health Policy Reason: Health policy is a set course of action undertaken by governments or health care organizations to achieve a particular health outcome. REF: p. 398

Chapter 22 91. According to the national Patient Safety goals, how can nursing reduce the risk of patients developing associated infections? (Select all that apply.)

a. Implementing current WHO hand washing guidelines b. Instituting sterile technique policies related to the care of central lines c. Creating mandatory competencies associated with the care of surgical incisions d. Striving to remove indwelling urinary catheters as soon as possible Reason: Safety goals to reduce the risk of health care-associated infections include complying with either the current Centers for Disease Control and Prevention (CDC) hand hygiene guidelines or the current World Health Organization (WHO) hand hygiene guidelines; implementing evidence-based practices to prevent health care-associated infections caused by multidrug-resistant organisms in acute care hospitals; implementing evidence-based practices to prevent central line-associated blood stream infections; implementing evidence-based practices for preventing surgical site infections; and implementing evidence-based practices to prevent indwelling catheter-associated urinary tract infections (CAUTIs). REF: p. 391; Box 22-2

Chapter 15 121. How does computerized provider order entry (CPOE) contribute to the prevention of medical errors? (Select all that apply.)

a. Improving communication between members of the health care team b. Providing support in the decision-making process c. Tracking of adverse patient focused events d. Providing real time checks of intervention related activities Reason: CPOE functions contribute to medical error prevention through improved communication, decision support, tracking and response to adverse events, and performance checks are done in real time. The CPOE may play a role in identifying appropriate diagnostic tests and preventing duplicate or contradictory orders for tests but it does not eliminate the human role associated with the monitoring results of such testing. REF: p. 269

Chapter 3 145. The nursing faculty incorporates which practice-based learning activity in order to meet the students' need to learn about the sociodemographics currently present among today's population?

a. Inclusion of a geriatric experience as a clinical requirement of all nursing students Reason: The sociodemographics of a population includes descriptions of the sociological and statistical information represented among a population. Age-related information would be included and thus bring attention to the growing population of older adults in the United States and their health needs. The political agenda of nursing organizations and health care delivery systems are considered political and economic rather that sociodemographic in nature. Language barriers would be considered cultural in nature. REF: p. 45-46

Chapter 7 27. Place the following events associated with health care finance in the appropriate chronological order. (Select all that apply.)

a. Individual disability and illness insurance policies become available b. Managed care emerges c. Medicare and Medicaid are created d. Blue Cross/Blue Shield originates at Baylor Hospital Reason: Individual disability and/or illness policies become available in 1890; First group health coverage is offered for a monthly charge; teachers in Dallas, Texas, contract with Baylor Hospital in 1929. This is the beginning of Blue Cross/Blue Shield insurance; In 1965 Medicare and Medicaid programs are created, making comprehensive health care available to millions of Americans; Between 1980 and 1990 Managed care plans emerged. REF: p. 109, Table 7-1

Chapter 15 118. Information is stored, retrieved, communicated, and managed using hardware and software that is collectively referred to as ________________________.

a. Information Technology Reason: Information technology is the term used to identify the hardware and software that enable information to be stored, retrieved, communicated, and managed. REF: p. 267

Chapter 23 15. A reliable private source of information about major health issues and health care reform is the:

a. Kaiser Family Foundation Reason: The Kaiser Family Foundation is a nonprofit, private foundation dedicated to providing trusted, independent information about major health issues facing our nation and is an excellent resource for reliable information about health care reform REF: p. 409

Chapter 1 131. What factor is cited as contributing to the inability of America's schools of nursing to meet the expected demand for professional nurses? (Select all that apply.)

a. Lack of school funding b. Inability to fill faculty positions c. Lack of clinical preceptors d. Insufficient clinical practice sites Reason: AACN's report on the 2013-2014 enrollments in Baccalaureate and Graduate Programs in Nursing, U.S. nursing schools turned away 78,089 qualified applicants from baccalaureate and graduate nursing programs in 2013 due to budget constraints and insufficient numbers of faculty, clinical sites, classroom space, and clinical preceptors. Budget constraints, an aging faculty, and increasing job competition from clinical sites have contributed to this emerging crisis (AACN, 2014). REF: p. 18-19

Chapter 25 68. Consider the roles during a meeting. Match the role with the responsibility

a. Leader i. 2- Calls the meeting to order b. Timekeeper i. 3- Assures that the meeting is kept on schedule c. Recorder i. 1- Creates the meeting's minutes

Chapter 23 20. How does health policy become law? (Select all that apply.)

a. Legislative action b. Executive directive c. Judicial decision Reason: The development of health policy occurs by enactment of legislation and the accompanying rules and regulations that carry the weight of law; administrative decisions/directives made by various governmental agencies of the executive branch; judicial decisions that interpret statutes, regulations, and settle legal disputes/conflicts within society. The opinion of the people is considered but does not initiate law while lobbying is the process that attempts to influence the legislative, executive, and/or judicial branches of government to enact a specific law. REF: p. 402

Chapter 6 100. Match the hierarchy of design levels with the forms of probing with each level.

a. Level I i. 2- Review of nonrandomized trails b. Level III i. 3- Review of correlation/observational studies c. Level V i. 1- Systematic review of descriptive/qualitative/physiologic studies d. Level VI i. 5- Single descriptive/qualitative/physiologic study e. Level VII i. 4- Opinions of authorities, expert committees

Chapter 23 17. Match the Term with its definition

a. Lobbying i. 2- Persuading, educating and/or convincing b. Platform i. 3-Statement of principles and policies c. Constituent i. 1-A person who is legible to vote in federal election d. Stakeholder i. 4- Individual who has an interest in and may be affected by policy

Chapter 2 137. The nursing profession viewed the television series _________ as a negative portrayal of nurses based on the exploits of the program's nurses and physicians.

a. MASH Reason: The nursing profession viewed MASH as professionally destructive because of the negative portrayal of Hot Lips Hoolihan and the nurses of the 4077th Army MASH (Mobile Army Surgical Hospital) unit in Korea. The sexual exploits of nurses and physicians and the uncaring Margaret provided few positive images. REF: p. 26

*Chapter 8 37. ____________ occurs when a professional fails to meet the standard of conduct required of all similar professionals.

a. Malpractice Reason: Malpractice is the failure of a professional to meet the standard of conduct that a reasonable and prudent member of his or her profession would exercise in similar circumstances that result in harm. The professional's misconduct is unintentional. REF: p. 120

Chapter 2 138. Match the nursing figure with the appropriate description.

a. Maria Ortiz i. 4- Died in a clinic bombing in Baghdad, Iraq b. Jennifer Moreno i. 1- Won a bronze medal for action in Afghanistan c. Florence Nightingale i. 3- Patient advocate for Crimean soldiers d. Edith Cavell i. 2- Executed by Germany during World War I

Chapter 15 117. _______________ is the term used to define the capabilities and standards that an electronic health records system must be capable of utilizing in order to qualify for financial incentives provided by Medicare.

a. Meaningful Use Reason: "Meaningful Use" is a defined set of EHR capabilities and standards that EHR systems must meet to ensure that their full capacity is realized and for the users (hospitals and physician/provider practices) to qualify for financial incentives from Medicare. REF: p. 267

Chapter 7 22. When an unemployed, blind adult requires medical treatment but cannot afford the services, the nurse suggests that the client is possibly eligible for:

a. Medicaid Reason: Medicaid is a federally funded insurance plan for low-income persons who are aged, blind, disabled, or for certain families with dependent children. REF: pp. 113-114

Chapter 7 28. Which statement is true when considering characteristics of a preferred provider organization (PPO)? (Select all that apply.)

a. Member pays a premium for a fixed percentage of expense covered b. Includes deductible and copayment c. Member may select physician d. Members pay less for physicians and facilities on the plan's preferred list Reason: Members do pay a premium for a fixed percentage of expense covered. The plan does include deductibles and copayments. Member may select physician but they pay less for physicians and facilities on the plan's preferred list. Preventive care may or may not be covered depending of the plan. REF: p. 110, Table 7-2

Chapter 26 173. Which statement concerning nursing demographics is true?

a. Most nurses practicing today have bachelor's degree in nursing. Reason: The U.S. Health and Human Services Administration issued a report in April 2013 stating that 55% of practicing nurses have a BSN. The other options are not true statements. REF: p. 455

Chapter 19 60. What observations made by the nurse manager, indicates that a nurse demonstrated positive conflict management interventions when a patient's family member angrily accuses the nurse of neglecting the patient's need to be ambulated more frequently? (Select all that apply.)

a. Moved the discussion to the privacy of the patient's room b. Kept culturally appropriate eye contact with the family member while discussing the issues c. Kept refocusing the discussion to the patient's needs associated with ambulation Reason: Positive conflict management interventions include those that prevent the situation from escalating while being open to the family member's concerns. Providing privacy, showing attention by keeping eye contact when culturally appropriate, and focusing on the issue identified as the problem will contribute to the resolution of the conflict. REF: p.348 , Box 19-4

Chapter 4 166. The mobility of nurses and the growing area of telehealth have resulted in the development of a new licensing model called the ___________________________.

a. Mutual Recognition Model Reason: In 1997, the Delegate Assembly of the NCSBN moved to a new level of nursing regulation. The assembly approved a resolution endorsing a mutual recognition model of nursing regulation. REF: p. 70-71

Chapter 4 167. Uniform testing for entry into professional nursing practice is accomplished through the _______________.

a. NCLEX-RN® examination Reason: Efforts to provide common definitions of nursing practice, equivalent educational standards for practice, and uniform testing for entry into practice through the NCLEX-RN examination have been very successful. REF: p. 70-71

Chapter 27 190. What topics are not appropriate for discussion during an initial employment interview? (Select all that apply.)

a. Nationality b. Sexual preference c. Credit status Reason: Topics either legal or appropriate for discussion during an initial job interview include nationality, sexual preference, and credit status while discussing reasons for leaving previous jobs and any criminal convictions are appropriate and legal. REF: p. 475, Box 27-5

Chapter 2 139. Match the source of funding with its identifying description

a. Nurse Reinvestment Act i. 2- Faculty Loan cancellations b. Patient Protection and Affordable Care Act i. 4- Increased limits for nursing student loans c. The Nurse Training Act i. 3- $250,000,000 was allocated for nursing education d. Campaign for Nursing's Future i. 1- $50,000,000 to focus enhancing the image of nursing

Chapter 26 177. Which nurse is identified as demonstrating advanced practice skills and knowledge? (Select all that apply.)

a. Nurse practitioner (NP) b. Certified nurse-midwife (CNW) c. Certified registered nurse anesthetist (CRNA) d. Clinical nurse specialist (CNS) Reason: The term advanced practice nurse (APN) includes nurse practitioners (NPs), certified nurse-midwives (CNMs), certified registered nurse anesthetists (CRNAs), and clinical nurse specialists (CNSs). While possessing excellent nursing skills and knowledge, the clinical nurse leader is not considered an advanced practice nurse. REF: pp. 461-462

Chapter 23 19. Examples of state health policies would include: (Select all that apply)

a. Nursing licensure b. Medicaid programs c. Health insurance exchanges d. Children's Health Insurance Program (CHIP) Reason: Health policy at the state level has a powerful influence on the health and safety of each state's residents that include the licensing of nurses, management of Medicaid funds, health insurance exchange programs, and the CHIP program. Smoke-free zones are determined by local health policies. REF: pp. 400-401

Chapter 3 150. Interprofessional care requires improvement in respect to which patient care area? (Select all that apply.)

a. Safety b. Patient-centered focus c. Efficiency d. Impartiality Reason: In response to the paradigm shift to interprofessional care, and providing care following the IOM's six aims for improvement; safe, effective, patient centered, timely, efficient, and

Chapter 20 80. Which statements reflect common nurse practice acts policies? (Select all that apply.)

a. Only nursing tasks can be delegated, not nursing practice b. The RN can delegate only what is within the scope of nursing practice c. The RN must evaluate patient outcomes resulting from the delegated activity d. The LPN works under the supervision and direction of the RN Reason: The common policies reflected in nurse practice acts include: only nursing tasks can be delegated, not nursing practice, the RN can delegate only what is within the scope of nursing practice, the LPN or LVN works under the direction and supervision of the RN, the RN must evaluate patient outcomes resulting from the delegated activity. The LPN or LVN and UAP do not practice professional nursing. REF: p. 355, Box 20-1

Chapter 28 197. In order to be prepared to answer NCLEX items, the tester should be familiar with what question formatting? (Select all that apply.)

a. Ordered response b. Fill-in-the blank calculations c. Chart exhibits d. Audio clips Reason: Candidates will receive test items in a variety of formats which include traditional multiple choice, multiple response, ordered response, fill-in-the blank calculations, and/or hot spots. Recently items may also include charts, graphics, sound, video, tables, and multimedia. Short essay items are not included in the examination. REF: p. 489

Chapter 27 186. An ___________________ includes activities that enhance one's ability to adjust to and function within a new environment.

a. Orientation Reason: An orientation involves activities that enhance adaptation to a new environment. REF: p. 468

Chapter 22 88. Match the appropriate attitude with its QSEN competency.

a. Patient-centered care i. 3- Encourage the patient to express personal values and needs b. Safety i. 1- Value the contribution of standardization to patient protection c. Team Work and collaboration i. 4-Appreciate the risks involved with the transfer of care d. Quality Improvement i. 2- Appreciate the value of how care can be improved by both individuals and teams

Chapter 1 122. The Mosaic Code is one of the oldest sources of what type of health promotion?

a. Personal and public hygiene Reason: Hebrews developed a system of laws called the Mosaic Code. This code, one of the first organized methods of disease control and prevention, contained public health laws that dictated personal, family, and public hygiene. The Egyptian culture is credited with the first established documentation of events, the use of over 700 drugs for the management of health and the use of midwives for the delivery of babies. REF: p. 3

Chapter 6 94. When researching an aspect of human grief response, the nurse researcher will likely select which method of qualitative design?

a. Phenomenology Reason: Phenomenology is designed to provide understanding of the participants' "lived experience." This method is a valuable approach for studying intangible experiences, such as grief, hope, and risk taking. REF: p. 95, Table 6-2

Chapter 7 26. Match the type of health insurance with the correct descriptor.

a. Point of Service (POS) i. 3-Allows use of providers outside the plan's preferred list or network, but requires higher premiums and copayments for services b. Health Maintenance Organization (HMO) i. 1-Referred for treatments, specialists, and services by the primary care physician c. TRICARE i. 2-Offered by Military Health Services System d. Medicaid i. 4-Covers long-term care for qualifying individuals

Chapter 2 141. Which intervention is associated with a focus of The Institute of Medicine's The Future of Nursing report? (Select all that apply.)

a. Preparing more nurses with advanced nursing degrees b. Facilitating nurses in practicing to the full scope of their educational preparation c. Supporting the nursing role in the interdisciplinary health care team d. Improving data collection to support nursing workforce planning Reason: The Institute of Medicine (2010) issued a landmark report, the Future of Nursing that has significantly impacted the image and future of nursing. This report included four main recommendations: (1) higher levels of nursing education (80% BSN by 2020 and doubling the number of doctorates) through seamless academic progression; (2) removal of practice barriers to enable nurses to practice to their full scope of practice based on their educational preparation; (3) inclusion of nurses as full partners with physicians and other professionals in redesigning the U.S. health care system; and (4) improved data collection and information infrastructure to facilitate more effective workforce planning and information infrastructure. REF: p. 28-29

Chapter: 24 153. Which nursing care model is most likely to support the novice nurse's familiarity with providing holistic nursing care?

a. Primary Reason: Most novice nurses are more comfortable with the whole-task system because it is more consistent with what they were taught in school. The whole-task system requires complete knowledge and encourages loyalty to the profession. The model of nursing care consistent with the whole-task system is primary nursing, in which the nurse is responsible for all the needs of the patient. The other options tend to fragment the care and divide it among many other providers. REF: p. 420

Chapter 15 112. A personal decision to withhold information is considered the right to:

a. Privacy Reason: Privacy refers to the right of an individual to keep information about himself or herself from being disclosed to anyone else. REF: p. 271

Chapter 15 119. Match the following security-protection focused terms with the appropriate description.

a. Privacy i. 1- The right to keep information about oneself from being disclosed to anyone else b. Confidentiality i. 3- The act of limiting disclosure of private matters c. Security i. 2- The means to control access and protect information from accidental or intentional disclosure

Chapter 17 9. Which situations have been identified as positive factors in achieving job satisfaction among nurses? (Select all that apply.)

a. Professional development opportunities made available at the facility b. Attention is paid to fostering excellent nurse-physician relationships c. Respect is given to and expected by all members of the health care team Reason: Nursing job satisfaction depends on attention being paid to professional development and advancement opportunities, support for good nurse-physician cooperation and a pleasant respectful work environment. A lack of collaborative decision making and a management team uninvolved in the daily function of the nursing units serve as negative factors. REF: p. 305, Box 17-5

Chapter 27 189. Which documents are appropriately included in a professional nursing portfolio? (Select all that apply.)

a. Proof of an advanced cardiac life support (ACLS) certification b. Continuing education certificates c. Copies of poster presentations d. Letters of recommendations Reason: A portfolio appropriately can contain proof of professional certifications; educational credentials that include earned CEUs; scholarly activities like poster presentations; letters of recommendations. The focus of a portfolio is past and current accomplishments not the plans for future ones. REF: p. 474, Box 27-4

Chapter 3 142. Which educational intervention demonstrates the most acceptable application of technology as a trend that is currently influencing nursing education as well as client safety?

a. Providing a list of acceptable Internet sites to use as resources for medication administration Reason: The ever-expanding need for rapid, accurate information has presented a problem for both faculty and student. Appropriate Internet sites can serve as effective sources of information when faculty plays a role in identifying these sites. Such faculty involvement also helps ensure client safety because this information is used to provide client care. Scheduling time for the students to become familiar with computer-based documentation systems is helpful but is not as safety oriented as is the correct option. Requiring personal computers for classroom work and testing and allowing electronic submission of written assignments is student focused rather than client focused. REF: pp. 40-41

Chapter 6 101. Match the names and descriptions of the following research designs

a. Quasi-experimental research i. 1- Design that lacks one of the components of an experimental design b. Phenomenology i. 3- Inductive descriptive methodology to describe the lived experiences of study participants c. Methodologic design i. 4- Used to develop the validity and reliability of instruments that measure research concepts and variables d. Grounded theory i. 2- Used to collect and analyze data aiming to develop theories grounded in real-world observations

Chapter 20 78. Which situations demonstrate effective delegation by the RN to the LPN? (Select all that apply.)

a. RN asks the LPN, "Do you have any concerns about your assignment for today?" b. LPN asks, "Where will you be if I need help with the dressing I've been assigned?" c. RN adjusts the LPN's assignments when a patient's condition becomes unstable d. RN assumes responsibility for ambulating a patient when he reports, "I'm feeling dizzy." Reasons: The RN is responsible for communicating effectively with the LPN concerning responsibilities and accountability regarding patient care. Asking the LPN if she/he has any concerns demonstrates effective communication. Appropriate supervision is demonstrated by the LPN's when a question about support is asked. Evaluation and reassessment is demonstrated by the RN adjusting assignments and assuming a responsibility when the patients' conditions change. REF: p. 360, Box 20-4

Chapter 7 30. What area of professional nursing is directly affected by changes in financing of health care services? (Select all that apply.)

a. Rate of reimbursement for nurse practitioners b. Job opportunities for salaried health care providers c. The public's willingness to seek services for various health care providers d. The general perception of the worth and values of health care services Reason: The rules of payment are important as a reflection of the value and worth society places on health care services for the public. Government policy influences the public's openness to secure services from various professionals such as nurse practitioners. Financing affects salaried employees because health care providers build job opportunities based on payment sources. Payment modes will determine whether a particular nursing role will be reimbursed, affecting specialties and professional autonomy. Health care accessibility is not considered directly influenced. REF: pp. 115

Chapter 28 199. When performing a personal needs assessment as preparation for taking the NCLEX, testers should consider what recourses? (Select all that apply.)

a. Reports on nationally normed assessment examinations b. Rationales provided for review items c. Nursing faculty members d. Previous examination grades from nursing courses Reason: Look at previous academic and nursing coursework, class notes and handouts, examination grades, and grades made on nursing care plans or process papers. If your school uses nationally normed assessment examinations, use the report to determine areas of strengths and weaknesses. The rationale provided when reviewing these exams provides a wealth of information and validates the correct answers. Talk with faculty in courses in which weakness or difficulty existed. Talking with those who were successful does little to provide you with insight concerning your own personal strengths and weaknesses. REF: p. 493-494

Chapter 1 125. Which of the following interventions best demonstrates that a nursing administrator understands the challenges currently facing the professional nurse regarding patient safety?

a. Requiring that staffing and schedules accommodate the attendance at mandatory in-services focusing on interprofessional care collaboration Reason: The changing duties, responsibilities, and conflicts amid nursing shortages and public concern over patient safety and quality of care characterize present-day practice. These changes require professional nurses to have core competency in critical thinking and communication as well as interprofessional collaboration, assessment, leadership, and technical skills, in addition to their knowledge of health care. Although nurses support providing holistic care to both client and family and providing for appropriately designed patient rooms and common areas, interprofessional care collaboration has a much greater impact on the delivery of appropriate and safe client care. Although client advocacy is a nursing responsibility, the focus in this situation is on client autonomy, not safety. REF: p. 18-19

Chapter 3 143. Increasing professional and personal responsibility has been identified as an issue influencing the education of today's nursing professional. Which of the following has the greatest impact on client safety in the acute care setting?

a. Scheduling mandatory competency assessments on all shifts to facilitate compliance Reason: Although all the options contribute to safe nursing care, regular, rigorous assessment of clinical skills has the greatest impact on client safety. REF: p. 51

Chapter 9 42. Normally, by what developmental milestone is a person expected to have learned there are consequences for making bad decisions?

a. School age Reason: By the time children reach school age, they have learned that behavior has consequences and that good behavior is associated with rewards and bad behavior with punishment. Through their experiences and social interactions with people outside their home or immediate surroundings, school-age children begin to make choices about how they will act based on an understanding of good and bad REF: p. 168

Chapter 19 57. Anyone who wishes to convey an idea or concept to others is referred to as a _________________.

a. Sender Reason: Sender is anyone who wishes to convey an idea or concept to others, to seek information, or to express a thought or emotion. REF: p. 329

Chapter: 24 160. What characteristics are generally possessed by effective mentors who work with novice nurses? (Select all that apply.)

a. Sensitive to the needs of others b. Welcomes change c. Possesses moral and ethical integrity d. Is nonjudgmental Reason: The characteristics that are demonstrated by effective mentors include sensitivity to others, flexibility, demonstrate moral and ethical integrity, and is nonjudgmental. Management skills would be important only if you were seeking to develop management skills. REF: p. 426, Box 24-4

Chapter 22 89. Which actions demonstrate a nurse's understanding and implementation of QSEN's competencies? (Select all that apply.)

a. Shares a patient's religious food preferences with members of the health care team b. Asks for assistance when transferring a large patient onto a stretcher c. Encourages a patient newly diagnosed with terminal cancer to talk about their end-of-life concerns d. Attends an in-service on a piece of equipment new to the unit Reason: QSEN competencies are demonstrated in the sharing of patient preferences and encouraging the patient to express their needs and values (Patient-Centered Care); Asking for assistance from team members (Teamwork and Collaboration); Learning to use equipment properly (Safety). Deviating from policies and practices based on evidence-based practice without first consulting clinical experts on the subject is clearly not demonstrating EBP. REF: p. 394, Table 22-1

Chapter 6 97. Research that places importance on the identification of best practices and benchmark innovation is the focus of the nursing organization ______________________.

a. Sigma Theta Tau International (STTI) Reason: Sigma Theta Tau International (STTI) makes research grant awards to increase scientific knowledge related to nursing practice. STTI supports creative interdisciplinary research and places importance on identifying "best practices" and benchmark innovations. REF: p. 93

Chapter 22 85. Which example of discharge teaching addresses a never event as identified by The Centers for Medicare & Medicaid Services?

a. Signs and symptoms of both hypo- and hyperglycemia Reason: The manifestations of poor glycemic control has been identified as a never event by The Centers for Medicare & Medicaid Services. REF: p. 392, Box 22-3

Chapter 16 209. Which body systems are likely routes for chemical agents to cause injuries among a large number of people? (Select all that apply.)

a. Skin b. Nervous c. Circulatory d. Respiratory Reason: Chemical agents injure or kill through variety of means: vesicant (skin), nerve (nervous system), blood (circulatory), and respiratory. While the kidneys may ultimately be affected, the urinary system is not a primary route of injury. REF: p. 284, Table 16-1

Chapter 16 207. Those affected with the H1N1 virus were encouraged to stay home for 24 hours after their fever had subsided. This is an example of a form of a control measure called ______________________.

a. Social distancing Reason: The term social distancing refers to the attempt to keep people as far apart as possible to limit the possibilities of spreading germs. The most frequently recommended type of social distancing that has proved to be effective for H1N1 virus has been keeping people suspected of the illness at home until 24 hours after the fever has subsided. REF: p. 291

Chapter: 24 154. In order to best ensure that a novice nurse will transition well into the role of clinical care provider, the emphasis initially should be focused on:

a. Socialization Reason: The transition from student to professional nurse is difficult. Socialization of the novice nurse is key to his or her ability to transition or just "survive" at the clinical level. The other options are relevant, but not initially. REF: p. 421

Chapter: 24 157. Match the following terms with the appropriate definition.

a. Socialization i. 2- The nurturing and integration of a person into the profession of nursing b. Trust i. 1- Confidence that your peers have good intentions toward you c. Inuition i. 4- Extracting clues from one's subconscious d. Mentoring i. 3- A mutual interactive method of learning

Chapter 26 172. A nurse, acting as the coordinator of the interprofessional health care team of a client who recently experienced a stroke, will consult with which member of the team to help address the client's problem with swallowing?

a. Speech-language pathologist Reason: The speech-language pathologist assists clients who are communicatively impaired by intervening in speech, language, and/or swallowing disorders. The dietitian provides nutritional therapy and support to ensure that the nutritional needs of the client are met with consideration to any problems the client may have with swallowing. The respiratory therapist is responsible for assessment and maintenance of the client's airway and respiratory equipment used for diagnosis and therapy of respiratory disorders. The occupational therapist plans activities that assist and teach clients with physical disabilities to become independent in activities of daily living, such as dressing, grooming, bathing, and eating. REF: p. 452

Chapter 1 128. The first diploma nursing program for blacks is established at _______________.

a. Spelman College, Atlanta, Georgia Reason: Spelman College, Atlanta, Georgia, established the first diploma nursing program for blacks in 1886. REF: p. 4, Box 1-1

Chapter 15 120. Match the following electronic health record's "Meaningful Use" stages with their appropriate criteria.

a. Stage 1 i. 2- The capture and access of patient health data to be exchanged among health care teams b. Stage 2 i. 1- The electronic exchange of diagnostic test results c. Stage 3 i. 3- Utilizing electronic health record information to improve patient outcomes

Chapter 10 109. According to the Standards of Practice for Culturally Competent Nursing, match the following standards with the appropriate description.

a. Standard 2: Critical Reflection i. 3- Have an awareness of how personal beliefs can affect culturally congruent nursing care b. Standard 4: Culturally Competent Practice i. 1- Implement culturally congruent nursing care c. Standard 10: Cross Cultural Leadership i. 4- Have the ability to influence diverse populations d. Standard 11: Policy Development i. 2- Have the knowledge and skills to work with public and private organizations, professional associations, and communities

Chapter 10 108. Believing all Muslims are terrorists is an example of _____________________.

a. Stereotyping Reason: Stereotyping occurs when one assigns certain beliefs and behaviors to groups without recognizing individuality. REF: p. 180

Chapter 27 187. When considering a cover letter, in what order should the following information be addressed? 1 being first and 4 being the last.

a. Suitability b. Interview c. Purpose d. "Handshake" Reason: Paragraph 1 is a statement of purpose identifying the position you are interested in; paragraph 2 emphasizes your suitability for the desired position; paragraph 3 is a request for an interview; lastly a "written shake" is included. REF: p. 471-472

Chapter 20 76. The active process of directing, guiding, and influencing the outcome of an individual's performance of an activity or task is referred to as _______________________.

a. Supervision Reason: Supervision is the active process of directing, guiding, and influencing the outcome of an individual's performance of an activity or task. REF: p. 352

Chapter 1 130. In order to meet the projected need for professional nurses, what area of concern is funded by the Nurse Reinvestment Act of 2002? (Select all that apply.)

a. Support for those enrolled in master's and doctoral nursing programs b. Recruitment enticements for nursing faculty positions c. The establishment of retention programs focused on existing nursing faculty Reason: Public funding to schools of nursing needs to be increased to attract and retain nursing faculty. In support of the nursing profession, the U.S. Congress adopted the Nurse Reinvestment Act to provide funds for nursing education, recruitment, and retention programs. President George W. Bush signed the bill into law in August 2002. Increased classroom space for associate and baccalaureate programs as well as higher compensation for clinical nurses are not foci of this legislation. REF: pp. 18-19

Chapter 22 86. A widely used approach of encouraging cooperation between the health care professions is the use of:

a. Team Strategies and Tools to Enhance Performance and Patient Safety (Team STEPPS). Reason: A widely used team training approach for health care teams is Team STEPPS (Team Strategies and Tools to Enhance Performance and Patient Safety). Team STEPPS acknowledges that team training and enhanced communication are among the essential components of a comprehensive patient safety system.

Chapter 3 149. What patient care area is addressed by QSEN Competencies? (Select all that apply.)

a. Team work b. Informatics c. Quality Improvement d. Evidence-based practice Reason: The QSEN competencies at both the prelicensure (2007) and graduate level (2012) assess the knowledge, skills, and attitudes of nurses within six areas: patient centered care, teamwork and collaboration, evidence based practice, quality improvement, safety, and informatics. Nursing professionalism is not an addressed area. REF: pp. 43-44

Chapter 1 123. What event in American history influenced the role black women have played in the nursing profession?

a. The Civil War, beginning in 1861 Reason: The American Civil War allowed and encouraged black women to volunteer as nurses, thus setting the stage for their ultimate involvement in the profession of nursing. The U.S. Marine Hospital Service was the forerunner of today's U.S. Public Health Service. The Army Nurse Corps was an outcome of the Army Reorganization Act. North Carolina was the first state to implement the registration of nurses through its nursing practice act. REF: pp. 9

Chapter 16 204. Which agency should be contacted during the preparedness phase to help provide guidance in putting into place a plan to ensure all involved agencies work flawlessly during a disaster?

a. The National Incident Management System (NIMS) Reason: NIMS provides a systematic proactive approach to guide departments and agencies at all levels of government, as well as nongovernmental organizations and the private sector, to work seamlessly during disaster situations. The CDC host a Strategic National Stockpile (SNS) that has large quantities of medicine and medical supplies to protect the American public if there is a public health emergency severe enough to cause local supplies to be depleted. DMATs are regionally organized teams consisting of physicians, nurses, and other health care providers that can be sent into areas outside their own regions to assist in providing care for ill or injured victims at the location of a disaster or emergency. FEMA coordinates federal support in the case of disasters. REF: pp. 288

Chapter 1 129. Place the following events related to professional nursing in correct order beginning with the initial event.

a. The Pennsylvania Hospital is established b. The New England Hospital opens a nursing program c. The Visiting Nurse Association is established d. The first issue of the American Journal of Nursing is published e. The Army Nurse Corps is established Reason: In 1751, the Pennsylvania Hospital is the first hospital established in America. In 1872, a school of nursing opens at the New England Hospital for Women and Children in Boston, Massachusetts. In 1886, the Visiting Nurse Association (VNA) is started in Philadelphia. In 1900, the first issue of the American Journal of Nursing is published. In 1901, the Army Nurse Corps is established under the Army Reorganization Act. REF: p. 4, Box 1-1

Chapter 4 162. A nursing student is telling peers about a scheduled appointment with a primary care provider. The student says, "One of the office staff come to take my blood pressure and said, "I will be your nurse today." "I noticed the name tag said nursing technician." Which of the following is the best response the student should make when explaining the implications of the nurse practice acts related to the technician's statement?

a. The RN title is defined and protected by the purpose of the nurse practice act. Reason: The first purpose of a Nurse Practice Act is to protect the health and safety of the public, the second purpose is to protect the title of RN. The legal title RN is reserved for those meeting the requirements to practice nursing. Only those licensed may use the designation of RN. Licensure requirements describe qualifications and procedures necessary for initial entry into nursing practice. The definition of nursing is of utmost importance because it delineates the scope of practice for nurses within the state, that is, each act outlines the activities nurses may legally perform within the jurisdiction. REF: pp. 65-67

Chapter 28 198. Which statement is correct when considering the value of NCLEX-RN review courses? (Select all that apply.)

a. The courses are applicable for graduates of all three types of nursing programs. b. A review course is no better than the person or people teaching it. Reason: Such courses are designed to reconsider or reexamine content common to the three types of nursing programs and the value of the course is dependent on the knowledge and skill of those teaching it. Just as there is no one best method for preparing to take the NCLEX-RN examination, there is no one best review course for a candidate to take in preparation for the examination. Review courses are not intended to teach totally new concepts. Review courses usually are expensive and last from 1 or 2 days to a complete week. REF: p. 495-496

Chapter 23 12. The nurse's role in the process of utilization review was a direct result of which federal health policy?

a. The creation of Diagnosis-Related Groups (DRGs) (1983) Reason: The introduction of DRGs forced hospitals to reduce patient stays and so a reduction in nursing staff occurred. New roles for nurses emerged, including that of utilization review. REF: p. 401, Table 23-1

Chapter 28 193. Which statement concerning the testing process regarding the NCLEX-RN examination is true?

a. The examination does not allow the skipping of questions Reason: Candidates taking the NCLEX-RN examination are not allowed to skip questions and return to them at a later time or to change answers to questions once an answer has been selected and entered into the computer. It is likely you will be expected to take alternatively formatted items while competency is based on items answered correctly. REF: p. 488-489

Chapter 4 163. Which of the following was an initial factor in the mutual recognition model of nursing regulation?

a. The growing presence of nurse-managed telehealth services Reason: Through this model, individual state boards formed the NLC, a collection or compact of states that recognize a nurse's right to practice in all involved states as long as they are licensed in at least one of the compact states. This form of regulation was motivated by the increased mobility of nurses, the telehealth movement, and the necessity of caring for patients across long distances. Concerns have emerged relating to monitoring nurses' practice in multiple jurisdictions, nurse privacy, and due process rights. REF: pp. 67/70

Chapter 19 51. The most likely reason a 9-year-old child cries and refuses to cooperate with an injection is:

a. the child's past experiences with injections. Reason: Previous experiences with injections are likely to have the greatest impact on the way a child will interpret any communication about an injection. REF: p. 331

Chapter 10 103. Which statement about racially or ethnically diverse populations is true?

a. They are often members of the lower socioeconomic class. Reason: Most families with racially or ethnically diverse backgrounds have a lower socioeconomic status than that of the population at large. There is no consistent evidence that these populations are generally unable to assume the American way of life. REF: p. 182

Chapter 25 62. Which statement demonstrates a problem in a nurse's attitude regarding the need to change?

a. This has always worked; it doesn't need to be changed." Reason: A person cannot affect a change on an experience if he or she is not willing to do something differently that could adjust the outcome. REF: p. 439-440

Chapter 9 50. What are the nurse's fundamental responsibilities as described by the International Council of Nurses Code of Ethics? (Select all that apply.)

a. To promote health b. To prevent illness c. To restore health d. To alleviate suffering Reason: The fundamental responsibility of the nurse is fourfold: to promote health, to prevent illness, to restore health, and to alleviate suffering REF: p. 165, Box 9-2

Chapter 10 104. Acceptance of others regardless of their cultural background is referred to as:

a. Tolerance Reason: Tolerance reflects an attitude that accepts differences. Respect is a demonstration of the inherent worth of the individual, regardless of differences. REF: pp. 183-184

Chapter 16 203. Which nursing action is seen during the relief response phase of a disaster?

a. Triaging patients brought to emergency treatment locations Reason: The relief response phase requires the interaction of emergency responders to be able to triage and provide assistance to the victims and stabilize the scene. The preparedness phase would require the other options. REF: p. 288

Chapter 6 99. Match the following terms associated with research with the appropriate description.

a. Triangulation i. 2- Variety of methods to collect data on the same concept b. Secondary analysis i. 3- Data previously collected for one study is analyzed for another study c. Practice Guidelines i. 4- Standards of practice, procedures, or decision algorithms d. Needs assessment i. 5- The researcher estimates the resource requirements of a group e. Abstract i. 1- Brief overview of a research study

Chapter 9 47. The customs and behaviors that society regards as desirable are called _____________.

a. Values Reason: Values are the ideas of life, customs, and ways of behaving that society regards as desirable. REF: p. 162

Chapter 8 34. In what situation may a nurse deliver care that is not considered at the level of required standard of care?

a. When the situation is determined to be an emergency Reason: Nursing care rendered in a life-threatening emergency may breach the standard of care required under ordinary circumstances. REF: p. 138

Chapter 20 73. Which factor has the greatest impact on whether safe delegation of nursing tasks is possible?

a. Whether the client's physical condition is considered stable at the time Reason: Generally the more stable the patient, the more likely delegation is to be safe. However, it is also important to remember many tasks that can be delegated may also carry with them a nursing responsibility. REF: p. 355

Chapter 10 102. An example of a member of a marginalized population would include:

a. a homeless lesbian reason: Marginalized individuals are those who live on the periphery of society often because they are stigmatized by society. Such populations would include the homeless and homosexuals. REF: p. 182

Chapter: 24 151. The benefit to health care providers resulting from their effective efforts to help minimize reality shock among novice nurses includes:

a. a reduction in staff turnover. Reason: Reality shock leads to stress, which can threaten the well-being of new nurses and result in physical illness and mental exhaustion, leading to disillusionment with their career and ultimately to absenteeism and turnover. Although the remaining options are desirable, they are not generally related to reality shock. REF: pp. 415-416

Chapter 28 195. The greatest percentage of items will address client needs related to:

a. a safe and effective environment. Reason: The safe and effective category is subdivided into 17% to 23% devoted to management of care and 9% to 15% focused on safety and infection control. Health and promotion and psychosocial integrity each are 6% to 12%, and the remaining items are focused on physiologic integrity. REF: p. 491-492

Chapter 28 194. The majority of the NCLEX-RN examination items focus on the evaluation of a graduate nurse's:

a. ability to relate nursing knowledge to needed nursing care. Reason: Because nursing requires application of knowledge, skills, and abilities, the majority of NCLEX-RN examination questions are written at the application or higher cognitive level and critical thinking skills. The verbs understanding, remembering, and comprehending reflect lower-level learning items. REF: p. 490

Chapter: 24 155. Nursing educational programs attempt to prepare the novice nurse to enter into clinical practice as a(n):

a. advanced beginner nurse. Reason: Advanced beginners are able to perform adequately and make some judgment calls based on experience; most novice nurses enter the workforce during this stage. Beginners are nursing students, while competent and proficient nurses usually emerge after a period of independent nursing practice. REF: p. 421

Chapter 7 23. The 2010 Patient Protection and Affordable Care Act (PPACA) projects that within the next 10 years:

a. all Americans will have health care insurance available to them Reason: On March 23, 2010, President Obama signed into law the Patient Protection and Affordable Care Act. In its broadest view, the PPACA is the plan for a comprehensive national health insurance program to provide funding for U.S. citizens and legal residents to secure health insurance beyond the current programs such as Medicare and Medicaid. REF: pp. 112

Chapter 26 175. It is true that advanced practice nurses:

a. are considered expert clinicians in the specialty area. Reason: All APNs make independent and collaborative health care decisions and engage in active practice as expert clinicians. APNs are educationally prepared through master's-level education to assume responsibility and accountability for the health promotion, assessment, diagnosis, and management of client problems, including the prescription of medication. REF: p. 461-462

Chapter 23 11. The most effective means for a nurse to impact the health and wellness of those being cared for at a local hospital is to:

a. be active in local politics by lobbying city policymakers on health and wellness issues. Reason: Local health policies are created by local politicians and apply only to those people who are residents of that local community, whereas health policy enacted at the federal level applies to all residents in the United States. REF: p. 400

Chapter 15 114. An important change that point-of-care technology has required of nurses regarding their clinical care of patients is:

a. becoming proficient users of the electronic tools available to them. Reason: Nursing today requires understanding and skill related to the variety of technological resources available to the nurse. Point-of-care technology is an essential component of nursing practice as it moves from practice that relies on memory to one that emphasizes continuous use of resources as they are needed. This means that nurses must transform from being technical experts to knowledge workers and rely on the ever-increasing and reliable computer memory versus the overburdened and fallible human memory. REF: p. 273

Chapter 4 165. Although requirements governing the practice of advance practice nursing may vary significantly, all states require:

a. certification in the specialty area. Reason: All states require evidence of certification in the specialty area. A master's degree is not required in all states. Requirements concerning supervisory and collaborative practices vary, as do requirements for years of practice in the specialty. REF: p. 71-72

Chapter 20 71. The most basic factor contributing to the effective supervision of unlicensed assistive personnel (UAP) is the nurse's:

a. confidence in his or her ability to delegate appropriately. Reason: Because RNs are becoming increasingly responsible for delegation and supervision in today's health care system, it is imperative that they have confidence in their delegation skills and understand the legal responsibility that they assume when delegating to and supervising licensed personnel and UAP. RNs must know what aspects of nursing and health care can be delegated. REF: p. 353-354

Chapter 27 185. The primary reason that an employer requests a nurse to provide nursing references is to:

a. confirm the nurse's ability to provide safe nursing care. Reason: The bottom line for any employer who provides health care services to the public is to ensure that its nursing staff practices safely. Recruiters are looking to uncover anything that would impair a nurse applicant's ability to provide safe nursing care, such as incompetence, unprofessional conduct, unreliability in attendance, chemical dependency, or a record of criminal activity. For screening, recruiters have four primary sources of information that include references. Although the reference may provide some insight into the other options, that is not its primary focus. REF: p. 482-483

Chapter 25 63. When setting priorities, the nurse who is faced with two urgent tasks will:

a. delegate. Reason: The seriousness of the situation requires appropriate delegation of the task. The other options fail to address the serious nature of the task. REF: p. 441

Chapter 6 95. Regarding nursing research, the role of the clinical nurse specialist (CNS) is best utilized when:

a. designing research studies. Reason: The CNS is in an ideal position to link research to practice by assessing an agency's readiness for research utilization, consulting with staff to identify clinical problems, and helping staff to discover, implement, and evaluate findings that improve health care delivery. REF: p. 97

Chapter 22 81. The nurse demonstrates an understanding of patient-centered care when:

a. discussing the client's wish to include herbal preparations to treat an illness. Reason: Patient-centered care involves providing care that is respectful of and responsive to individual patient preferences, needs, and values and ensuring that patient values guide all clinical decisions. REF: p. 381-382

Chapter 2 136. A staff nurse routinely finds it difficult to receive clarification on prescriptions and orders written by a particular member of the medical staff. When numerous attempts to deal directly with the physician fail, the nurse most appropriately manages the problem by:

a. documenting the problem in terms of client safety concerns and forwarding the information to both the nursing manager and chief of medical staff. Reason: The problem will best be addressed as a medical staff problem with appropriate documentation of the problem as it affects client safety being forwarded to both nursing and medical staff administration. Refusing to treat a particular physician's clients or discussing the situation with the clients themselves is not professional behavior and so not a viable option. Involving the entire nursing staff may seem effective but it merely identifies the situation as a nursing problem rather than what it truly is—a medical problem. REF: pp. 31-32

*Chapter 25 65. An important factor when determining the usefulness of electronic devices in managing the clinical setting is to consider their:

a. effect on workflow. Reason: The addition of an electronic device should improve workflow and processes, not hinder them or create cumbersome ones. Taking time to learn to use new technology correctly will pay off in the long run by increasing effective use of time.

Chapter 8 31. The public's right to expect to receive treatment that meets the standards of nursing care is protected by:

a. federal law Reason: Federal laws have a major effect on nursing practice, mandating a minimal standard of care in all health care settings that receive federal funds. REF: p. 122

Chapter 28 191. Achieving a passing grade on the NCLEX-RN® examination confirms that the novice nurse:

a. has the minimal competencies needed for entry-level nursing. Reason: The purpose of the NCLEX-RN examination is to protect the public from unsafe practitioners and to assist state board of nursing in determining candidates' capabilities for performing in entry-level RN positions. The other options are not the focus of the NCLEX-RN. REF: p. 487-488

Chapter 19 53. The primary goals of good relationship building and client care are best achieved when all members of the interprofessional care team:

a. have developed a trusting relationship. Reason: The need for trust in the health care setting is not limited to the nurse-patient relationship but rather pervades all working relationships. Care is more effective when the nursing team and the interprofessional team share the essential element of trust. REF: pp. 333-334

Chapter 22 84. Health care institutions are automatically approved by The Centers for Medicare & Medicaid Services (CMS) when they:

a. have earned The Joint Commission (TJC) accreditation. Reason: Health care organizations that are accredited by TJC automatically meet foundational approval from the CMS. The other options are not true regarding CMS approval. REF: p. 384

Chapter 27 182. The primary benefit of a well-written cover letter is to demonstrate the nurse's:

a. interest in this specific position. Reason: A convincing cover letter will show how this candidate is different and convey to the recruiter why he/she is the best fit for the position. The letter should also address why this institution is the applicant's first choice. The other options are secondary benefits of a concise, well-written cover letter. REF: p. 471

Chapter 23 13. The Department of Health and Human Resources is a part of the executive branch of the federal government and:

a. manages federally funded health care programs passed by Congress. Reason: The Department of Health and Human Resources is one of 15 executive departments that manages programs passed by Congress. The Judicial branch determines the constitutionality of policies. The Legislative branch has sole power to allocate funds and can override a presidential veto. REF: pp. 402

Chapter 3 144. A nurse manager recognizes which of the following as an example of a desirable practice-based competency outcome for a recently licensed registered nurse? The novice nurse:

a. meets orientation expectations 1 week ahead of schedule. Reason: The outcomes of practice-based competency relate to clinical practice. Demonstrating the ability to achieve the goals of the facility's orientation program 1 week ahead of schedule shows clinical expertise and professional confidence, both reflective of practice-based competency. An excellent attendance record and an interest in earning an advanced nursing degree reflect professionalism, and although effective communication and interpersonal skills are desirable, they are not considered among the necessary clinical care skills. REF: p. 43

*Chapter 8 35. A client who insists on leaving the hospital against medical advice (AMA) after being properly advised signs the necessary form. In order to avoid legal consequences regarding the client's decision, the nurse should: a. objectively document all the details of the situation thoroughly but concisely in the client's medical record. b. notify the nursing supervisor of the client's decision and that all reasonable efforts were made to dissuade the client from leaving. c. have a member of the security staff escort the client off the facility's property and into their private vehicle or public transportation. d. with the client's permission notify a contact person that the client is leaving the hospital against medical advice.

a. objectively document all the details of the situation thoroughly but concisely in the client's medical record. Reason: Almost all health care facilities have an AMA form that patients are asked to sign when they decide to refuse or discontinue ordered therapy or intend to leave the facility. The value of the document in countering a claim of negligence should the patient or family later sue will depend in great part on the quality of the nurse's charting

Chapter 27 183. The novice nurse prepares a résumé that focuses primarily on being a:

a. one-page summary of education and work experience. Reason: The résumé is the most appropriate format for the new graduate and will complement the cover letter by filling in important details about educational and work experiences. An effective résumé should compress education and employment history into an attractive, easy-to-read one-page summary. The cover letter serves to attract the interviewer and is a basis for scheduling an interview, while the curriculum vitae (CV) is a detailed document that describes education and work history. REF: pp. 472-473

Chapter 16 201. The pivotal factor that makes an event a disaster is that it:

a. overwhelms the local resources. Reason: A disaster condition is defined as a significant natural disaster or man-made event that overwhelms the affected state and necessitates both federal public health and medical care assistance. Neither the location nor cause is a factor in determining an event as a disaster. REF: pp. 283

Chapter 15 116. To best ensure effective use of point-of-care advanced technologies in the future:

a. potential clients must be well educated concerning the use and reliability of health care technologies. Reasons: Although some clients trust technology, others do not and may be reluctant to accept care that is driven by technology. Their fears and concerns must be addressed in order for new, more advanced technologies to be accepted and utilized to their fullest. The other options, while true, do not have as great an impact as client acceptance on the use of future technologies. REF: p. 274

Chapter 27 184. The most effective way to determine whether a position is a good match for an individual's strengths and goals is to:

a. prepare a list of questions to ask that will help provide information about the position. Reason: The graduate's task is to enter the interview prepared to answer and ask questions that will help determine whether this organization, with its available job opportunities, is a good match. Although changing positions is an option, too many changes in a short period of time can be viewed as a negative by employers. The other options may prove helpful, but they are directed toward identifying personal strengths, weaknesses, and goals rather than identifying a good match in a position. REF: p. 475

Chapter 7 24. When hospitals receive payment for services based on the pay-for-performance model, the nurses' role is to:

a. provide client care that helps ensure positive health outcomes. Reason: Nurses are at the center of helping hospitals and other health care organizations successfully manage these new payment mechanisms (pay-for-performance and newer events) by ensuring that health outcomes are achieved and errors are prevented. REF: pp. 113-114

Chapter 10 105. A nursing school best demonstrates an understanding of strategies that support recruitment of minority nursing students when it:

a. provides same-culture mentors from among faculty and student populations. Reason: Strategies such as mentoring by a same-culture professional are effective in recruiting and retaining minorities in nursing because this provides both role models and culturally sensitive support. REF: pp. 187

Chapter 7 25. An example of nursing's influence on outcome management through consumer empowerment would include:

a. providing a postsurgical client with education regarding the early signs of incisional infection. Reason: Educating the client regarding possible complications and so making early identification of such problems possible will positively affect care outcomes and help manage health care costs for this client. REF: p. 116

Chapter 6 93. If a nursing research project strives to discover new information concerning client care practices, this project is said to be a model of:

a. qualitative design. Reason: Qualitative research is designed for discovery rather than verification. It is used to explore little-known or ambiguous phenomena. REF: p. 95.

Chapter 25 64. When considering the establishment of a to-do list, it should be:

a. readily available and easily accessed. Reason: The list is effective only when it is available and easily accessed; otherwise, it is not likely to be used. REF: p. 442

Chapter 10 106. Cultural humility is achieved through:

a. regular self-evaluation of biases. Reason: We can all aspire to achieve cultural humility, which incorporates a lifelong commitment to self-evaluation and self-critique. The other options will have little impact without effective self-evaluation. REF: p. 188

Chapter 15 115. The most effective way to assess the accuracy of information obtained from the Internet is to:

a. research several different sources while comparing the information for similarities. Reason: Determine whether the information is accurate by looking for documentation and referencing. Compare information on the website with information from other sources. Review links that go from the website to other Internet resources. Ensure that the links are going to high-quality resources. Currency is better ensured on websites that are frequently updated with information that is less than 5 years old. It is restrictive to rely on only a few familiar sites, especially if they are related to commercial (.com) organizations. REF: p. 276; Table 15-3

Chapter 15 113. In the clinical setting an example of the meaningful use of electronic health records that has greatest impact on the delivery of care would include the nurse:

a. reviewing physical therapy notes before attending a patient's interprofessional care team meeting. Reason: Improved care coordination is one of the five priorities established as meaningful use standards. Sharing information between interprofessional health care team members would demonstrate that priority. REF: p. 272

Chapter 25 61. When focusing on maintaining emotional energy, the nurse should focus on personal:

a. self-confidence. Reason: Self-confidence, self-control, and interpersonal effectiveness are all key to emotional intelligence and energy. REF: p. 439

Chapter 3 148. When clinical settings are limited, faculty can best assess competence associated with entry level clinical skills by implementing ___________________ assessments.

a. simulation (Standardized Patient) Reason: The use of standardized patients as a competency-based summative assessment allows faculty to assess mastery of entry level clinical skills, some of which student may not have access to in the clinical setting. REF: p. 44-45

Chapter 9 44. A nurse demonstrates client-centered altruism when:

a. spending time each evening with a client who has expressed feelings of sadness and loneliness. Reason: Altruism is showing true concern for the welfare of another: giving a lonely client individualized, personal attention demonstrates altruism. REF: p. 169, Table 9-1

Chapter 20 74. After first having a strong understanding of the standards of practice that govern delegation, the registered nurse must know the:

a. the condition and needs of the patient whose care is being delegated. Reason: The RN must then know the client whose care is being delegated. The client's condition and stability must be determined before tasks and outcomes can be determined or consideration be given to the skill level of the staff being delegated to. REF: p. 358

Chapter 7 21. Starting in the 1960s, the rapid introduction of technically advanced health care diagnostics and treatment procedures contributed negatively to:

a. the cost of health care services. Reason: The negative result of such innovations was overuse of health services and rapid cost inflation for health care services. REF: p. 108

Chapter 16 205. In cases of natural disasters, nurses need to be prepared to handle clinical situations that result from:

a. the lack of electrical power. Reason: In the case of natural disasters like hurricanes, floods, and blizzards, staff needs to be prepared to cope with power outages and damage to the institution. The other options are not clinical and are rarely seen when preparations are well planned. REF: p. 289-290

Chapter 2 135. The ability to maintain appropriate professional boundaries and preserve nurse-client confidences has been most severely impacted by:

a. the misuse of social media forums by nurses. Reason: The ramifications of inappropriate posting of information by nursing professionals as well as nursing students via the various forms of social media has created serious problems affecting confidentiality and professional boundaries. State board of nursing take these breaches of nursing practice seriously, revoking nurses' license to practice and expelling students from nursing programs. Currently the courts have upheld legal decisions involving such actions. REF: pp. 27-28

Chapter 27 181. Novice nurses can find it most helpful to discuss nursing positions with their clinical instructor, especially when the discussion is related to:

a. their strengths and weaknesses regarding nursing skills. Reason: Although the clinical instructor may be able to provide information on all the options, he/she is best suited to help the novice nurse evaluate his or her clinical skills so they can be aligned with the needs of a particular nursing position. REF: p. 469

Chapter 2 132. The image that best serves today's profession of nursing is one that portrays:

a. trust and belief. Reason: The image most important to today's nursing professional is one that supports the public's need to trust and believe in the nurse caring for them. The individual actually providing the care often changes with each shift change and so the perception of the profession as a whole is vitally important. Although the other options represent positive traits, they all seem to be embodied in the image of trust and belief in the public's collective mind. REF: pp. 28-29

Chapter 20 75. When considering supervisory responsibilities, the registered nurse (RN) must recognize that:

a. when two RNs work together to move a client, neither nurse is in a supervisory position Reason: One RN is working with another RN in a collegial relationship, and neither RN is in the position of supervising the other. Each RN is responsible and accountable for his or her own practice. However, the RN in a supervisory or management position (e.g., team leader, charge nurse, nurse manager), as defined by the health care organization, will be in a position to supervise other RNs. REF: pp. 361-362

*Chapter 9 45. A nurse preserves a client's right to autonomy by: a. answering a client's questions about his prognosis truthfully. b. answering a client's questions so he can make an informed decision. c. providing medication to a trauma victim in a timely, appropriate manner. d. providing pain medication only after a needed surgical consent has been obtained.

b. answering a client's questions so he can make an informed decision. Reason: Autonomy is the preservation of a person's right to make his or her own decisions such as in the case of informed consent. REF: p. 170-171 Veracity: truthfulness even when the truth may cause the patient emotional pain. Beneficence: demonstrated by doing good, such as when providing pain relief. Nonmaleficence: often ethically challenging, as in the case of withholding pain medication until a necessary consent is obtained.

*Chapter: 24 152. Compassion fatigue is most likely among nurses working with the: a. elderly. b. homeless. c. mentally ill. d. culturally diverse.

c. mentally ill. Reason: Nurses who work in emotionally charged environments, such as hospice, emergency departments, and mental health settings, are likely to experience this reaction. Intensive ongoing losses such as those in oncology care make nurses vulnerable to burnout and compassion fatigue. REF: p. 417-418

*Chapter 4 161. The primary purpose of nursing licensure, now and in the past is to: a. prevent others from using the title nurse. b. demonstrate a specialized body of knowledge. c. protect the public. d. enhance recognition for the profession.

c. protect the public. Reason: Protecting the public is one of the essential purposes of the state board of nursing, which regulates licensure. DIF: Knowledge REF: p. 65


Related study sets

Academic Team Social Studies Study Material

View Set

Astronomy Midterm UCLA, Astronomy Final

View Set

4.2: some probability rules- compound events

View Set

N450 PrepU CH 40 Musculoskeletal

View Set

mktg 351: post purchase processes

View Set